Diễn Đàn MathScope

Diễn Đàn MathScope (http://forum.mathscope.org/index.php)
-   2014 (http://forum.mathscope.org/forumdisplay.php?f=177)
-   -   [VMO 2014] Bài 2 - Đại số (http://forum.mathscope.org/showthread.php?t=46325)

huynhcongbang 03-01-2014 11:27 AM

[VMO 2014] Bài 2 - Đại số
 
Bài 2.
Cho đa thức $P(x)={{({{x}^{2}}-7x+6)}^{2n}}+13$ với $n$ là số nguyên dương. Chứng minh rằng đa thức $P(x)$ không thể biểu diễn được dưới dạng tích của $n+1$ đa thức khác hằng số với hệ số nguyên.

hakudoshi 03-01-2014 11:51 AM

$P(x)>0$ nên $P(x)$ vô nghiệm. Do đó $n+1$ đa thức kia có bậc không bé hơn 2 và đều vô nghiệm.
Mặt khác $P(1)=P(6)=13$. ... Bí :!

DogLover 03-01-2014 12:08 PM

Lâu lắm mới thấy một bài đa thức khả quy.
Giả sử có thể phân tích được thành tích của $n+1$ đa thức $P_{i}(x)$ khác hằng hệ số nguyên.
Nhận xét 1: P(x) vô nghiệm do đó các đa thức $P_{i}(x)$ đều có bậc chẵn. Và vì P(x) có bậc 4n nên suy ra có ít nhất 2 chỉ số $i$ sao cho $P_{i}(x)$ có bậc 2, giả sử là $k$ và $l$.
Nhận xét 2: Với mọi $i$ thì $P_{i}(6)-P_{i}(1)$ chia hết cho 5

Cho $x=1$ và $x=6$ thì ta thu được $$P_{1}(1).P_{2}(1)...P_{n+1}(1) = P_{1}(6).P_{2}(6)...P_{n+1}(6) = 13$$
Suy ra $P_{i}(1)$ và $P_{i}(6)$ chỉ nhận giá trị trong ${-1,1,13,-13}$.
Mà $P_{i}(6)-P_{i}(1)$ chia hết cho 5 nên với mọi i thì $P_{i}(1) = P_{i}(6)$

Xét $i = k$ và $i = l$, rõ ràng không thể có trường hợp $|P_{k}(1)| = |P_{k}(6)|=|P_{l}(1)| = |P_{l}(6)| = 13$ nên hoặc $|P_{k}(1)| = |P_{k}(6)| =1$ hoặc $|P_{l}(1)| = |P_{l}(6)| =1$. Giả sử là $P_{k}$.
Khi đó có hai trường hợp xảy ra là $P_{k}(x) = (x-1)(x-6)+1$ hoặc $P_{k}(x) = (x-1)(x-6)-1$. Cả hai trường hợp này $P_{k}(x)$ đều có nghiệm, suy ra P(x) có nghiệm ==> vô lý.
Như vậy điều giả sử là sai. Và bài toán đã được chứng minh

hien123 03-01-2014 12:10 PM

Xét P(x+1) rồi dùng tiêu chuẩn eisteiner mở rộng ta suy ra có đúng n đa thức bậc 2 va 1 đa thức bậc 2n. Nhận xét mỗi đa thức bậc 2 phải có hệ số tự do chia hết cho 11. Suy ra $36^n+13$ chia hết cho $11^n$, mâu thuẫn.

DaiToan 03-01-2014 12:20 PM

Giả sử P(x) phân tích được thành tích của (n+1) đa thức khác hằng hệ số nguyên.
Vì P(x) vô nghiệm thực nên các nhân tử đó phải bậc chẵn. Nhưng bậc của P(x) bằng 4n nên phải tồn tại một nhân tử f(x) bậc 2 (có lẽ đây là mấu chốt!)
Có thể giả sử f(x) có hệ số của x2 là 1: $$f(x) = {x^2} + bx + c;{\rm{ }}b,c \in $ $. Vì f(x) vô nghiệm thực nên c>0.
Do $$\mathop {\lim }\limits_{x \to + \infty } f(x) = + \infty $ $ và f(x) vô nghiệm nên f(1)>0; f(6)>0.
Mà P(1)=P(6)=13 nên f(1), f(6) là ước dương của 13;
Nhưng f(6)-f(1) chia hết cho 5 nên f(1)=f(6)=1 hoặc 13
Từ f(1)=f(6) suy ra b=-7, suy ra $$f(x) = {x^2} - 7x + c$ $.
* Nếu f(1)=1 thì c=7, suy ra $$f(x) = {x^2} - 7x + 7$ $ có nghiệm thực (vô lí).
* Nếu f(1)=13 thì c=19, suy ra $$f(x) = {x^2} - 7x + 19$ $.
Ta có f(2)=9 và $$P(2) = {4^{2n}} + 13$ $.
Rõ ràng P(2) không chia hết cho 3 trong khi f(2) chia hết cho 3 (vô lí).
Vậy bài toán được chứng minh hoàn toàn.

quangvinht2 03-01-2014 02:03 PM

Vì P(x) luôn dương nên không có ước bậc lẻ. Giả sử P(x) phân tích thành tích n+1 đa thức monic bậc chẵn có tổng bậc là 4n nên tồn tại hai ước P1(x) và P2(x) bậc hai bất khả quy trên R. Vì cả hai monic nên luôn dương và P1(1).P2(1)|13, P1(6)P2(6)|13 nên chỉ có mấy khả năng sau
1/ P1(1)=P1(6)=1=> P1(x) có nghiệm thực (mâu thuẫn)
2/{P1(1);P1(6)}={1;13} => mâu thuẫn vì P1(6)-P1(1) là bội 5.
3/ P1(1)=P1(6)=13 thì P2(1)=P2(6)=1 mâu thuẫn giống phần 1.

huynhcongbang 03-01-2014 02:53 PM

2 Attachment(s)
Xin giới thiệu với mọi người lời giải của anh Cẩn, ý tưởng biến đổi đại số kết hợp với tính chất số học khá thuần túy.

[Only registered and activated users can see links. Click Here To Register...]

[Only registered and activated users can see links. Click Here To Register...]

thaygiaocht 03-01-2014 02:58 PM

Đến đoạn $P(1)=P(6)$ rồi như các lời giải ở trên rồi có thể viết $P(x)=(x-1)(x-6)+k$. Do $P(x)$ vô nghiệm, giải $\Delta >0$ được $k>\dfrac{25}{4}$, lại có $k|13$ nên $k=13$.
Thay vào $P(x)=x^2-7x+19$.
Do đó $6^{2n}=-6 (\mod 19)$, có thể chỉ ra điều này vô lý bằng quy nạp.

huynhcongbang 03-01-2014 04:22 PM

Bài 2 này có nét gì đó giống một bài cũ trong IMO Shortlist 1991.

[Only registered and activated users can see links. Click Here To Register...]

Cũng kiểu đa thức nguyên và yêu cầu chứng minh chặn trên cho số thừa số mà nó phân tích ra được. :)

namdung 04-01-2014 08:33 AM

Thực ra ta có thể chứng minh kết quả mạnh hơn như sau:

Nếu $P(x) = (x^2-7x+6)^{2n} + 13 $ có thể phân tích thành tích của 2 đa thức Q(x), S(x) với hệ số nguyên thì Q(x) và S(x) đều có bậc 2n.

Từ đây với n > 1 bài toán trở nên hiển nhiên, còn với n = 1 ta có thể kiểm tra trực tiếp rằng $(x^2-7x+6)^2 + 13 $ bất khả quy.

Thật vậy, giả sử P(x) = Q(x).S(x).

Gọi $x_1, x_2, ..., x_{4n} $ là các nghiệm phức của P(x) thì sẽ là tích của các thừa số $(x-x_i) $. Đánh số lại nếu cần, ta giả sử $Q(x) = (x-x_1)(x-x_2)...(x-x_k) $ với $1 \le k < 4n $.

Ta có $((x_i-1)(x_i-6))^{2n} = -13 $. Từ đây suy ra $|(x_i-1)(x_i-6)| = 13^{\frac{1}{2n}} $. (*)

Mặt khác $(1-x_1)...(1-x_k) = Q(1) $ nguyên nên $|(1-x_1)...(1-x_k)| $ nguyên. Tương tự $|(6-x_1)...(6-x_k)| $ nguyên.

Từ đây suy ra $m = |(x_1-1)(x_1-6)(x_2-1)(x_2-6)...(x_k-1)(x_k-6)| $ nguyên.

Nhưng theo (*) thì $m = 13^{k/2n} $

Suy ra k = 2n. Vậy Q(x), S(x) đều phải có bậc là 2n.

Tôi có cảm nhận rằng đa thức P(x) là bất khả quy, tuy nhiên cần thời gian để kiểm tra chắc chắn.

Lời giải trên đây dựa trên ý tưởng của 2 bài toán kinh điển sau:

1. Tìm tất cả giá trị n sao cho đa thức $x^n + 4 $ có thể phân tích thành tích của hai đa thức khác hằng số với hệ số nguyên.

2. (IMO 1993) Chứng minh rằng với mọi n> 1, đa thức $x^n - 5x^{n-1} + 3 $ bất khả quy.

hung_020297 05-01-2014 10:43 AM

Nếu mà làm được đến đa thức bậc 2 thì có được điểm ko ạ

kien10a1 05-01-2014 11:10 AM

Trích:

Nguyên văn bởi namdung (Post 199176)

Tôi có cảm nhận rằng đa thức P(x) là bất khả quy, tuy nhiên cần thời gian để kiểm tra chắc chắn.

Em thấy như thầy Nam Dũng phân tích, thì ta chắc chắn có $\left | Q(1)Q(6) \right |=13 $
Vậy thì mọi chuyện xong rồi chứ nhỉ, chú ý thêm $ Q(1)-Q(6) \vdots 5 $ thì ta suy ra ngay không có đa thức Q hệ số nguyên thỏa mãn.

vickyjustice 05-01-2014 01:46 PM

Xet P(x+1) roi nhan xet neu phan tich dc thoa de bai thi co 2 da thuc bac 2 vo nghiem, he so tu do la 13 nen he so tu do cua 1 trong 2 da thuc do phai =1, vay chi co 3 truong hop: $x^2+1 $, $x^2+x+1 $, $x^2-x+1 $, thay nghiem phuc thay vo ly


Múi giờ GMT. Hiện tại là 10:37 AM.

Powered by: vBulletin Copyright ©2000-2024, Jelsoft Enterprises Ltd.

[page compression: 14.15 k/14.99 k (5.63%)]